Consider the following augmented matrix (3)


1 −1 2 1
3 −1 5 −2
−4 2 k
2 − 8 k + 2

 .
Determine the values of k for which the system has
(i) exactly one solution,
(ii) infinitely many solutions,
(iii) no solution.



Answer :

Other Questions